775,710 teenagers voted for their favorite singer on a TV show. Each singer received the exact same number of votes. How many singers could there have been?a) 5b) 2c) 6d) 9

Answers

Answer 1

There could have been 2,5,6, and 9 singers

1) Regarding 775,710 as an even number then for sure there could have been 2 singers on this TV Show

[tex]775710\colon2=387,855[/tex]

2) On the other hand, 775710 is also divisible by 5, for its last number is 0. So there could also have been 5 singers.

[tex]775710\colon5=155,142[/tex]

3) This number of teenagers can also be divided by 6 (for it is divisible by 2 and by 3) and by 9 as well ( the sum of the digits of 775710, 7+7+5+7+1+0=27 is also divisible by 9).

Hence, there could have been 2,5,6, and 9 singers since 775,710 can be divided exactly by 2,5,6, and 9.


Related Questions

Given the following table of values for f(x), find f(0).
x −3 −1 0 3 6 9
f(x) 1 8 1 0 13 −3

Answers

The value of the function at x = 0 will be 1.

What is a function?

A statement, principle, or rule that creates a relationship between two variables is known as a function. Functions are abundant in mathematics and are required for the creation of complex relationships.

When the required parameters and natural laws are given values, the expression yields the calculation result that the mathematical model represents.

The table represents a function.

x     f(x)

-3     1

-1     8

0      1

3     0

6     13

9     -3

In the above table, we can see that at the value of x = 0 in the function the value of the whole function is 1.

Let's learn more about the function

brainly.com/question/5245372

A toy rocket is shot vertically into the air from a launching pad 5 feet above the ground with an initial velocity of 32 feet per second. The height h, in feet, of the rocket above the ground at t seconds after launch is given by the function

h(t)=-16t²2 +32t+5.

How long will it take the rocket to reach its maximum height? What is the maximum height?

Answers

It takes the rocket to reach its maximum height h max = 17(ft).

The object's maximum height is the highest vertical position along its trajectory.

How to find a maximum height?The object's maximum height is the highest vertical position along its trajectory. Before reaching the highest point, the object is flying upwards and then falls. It means that the vertical velocity is equal to 0 at the highest point of projectile motion (v y = 0 v y = 0 v_y=0).

So, h(t) = -16+ 32t+5:

t max = time for maximum heightt max = 32 / 2*(-16)= 32 / 32 = 1h max = the maximum height above the groundh  max = h(1) = -16() + 32*1 +5-16+32+5 =  -16+ 37 = 21

Then, h max rocket = the maximum height of the toy rocket

h max rocket  = 21 -5 = 17(ft)t max = 1 secondh max = 17(ft)

Therefore, it takes the rocket to reach its maximum height h max = 17(ft)

The object's maximum height is the highest vertical position along its trajectory.

To learn more about maximum height, refer to:

brainly.com/question/12446886

#SPJ13

3.)The area of a regular polygon is 145.8 sq. cm. If the perimeter of this polygon is 108 cm,find the length of the apothem.

Answers

The area of the regular polygon is a= 145.8 sq.cm

Perimeter of the polygon is p=108 cm.

Let apothem length be a

We know,

a=p x a/2

Putting the values,

145.8=108xa/2

a=2.7 cm

The length of the apothem is 2.7 cm

Is (x+1) a factor of -2x^5-4x^4+x-10?

Answers

Notice that:

[tex]-2x^5-4x^4+x-10,[/tex]

cannot be expressed as a product of factors, meaning that

[tex](x+1),[/tex]

is not a factor of the given polynomial.

Answer: Not a factor.

If theta is in quadrant II and cos theta= -3/5, what is sin 2 theta + cos 2 theta?

Answers

ANSWER

sin(2θ) + cos(2θ) = -31/25 = -1.24

EXPLANATION

If we do the inverse of the cosine to -3/5 we would get the angle θ. Then we can know the value of the sine:

[tex]\sin \theta=\sin (\cos ^{-1}(-\frac{3}{5}))=\frac{4}{5}[/tex]

So we have:

• sin(θ) = 4/5

,

• cos(θ) = -3/5

To find sin(2θ) + cos(2θ) we'll have to use the trigonometric identities:

[tex]\begin{gathered} \sin 2\theta=2\sin \theta\cos \theta \\ \cos 2\theta=1-2\sin ^2\theta \end{gathered}[/tex]

Since we have the sine and cosine of theta, we can solve this:

[tex]\sin 2\theta=2\cdot\frac{4}{5}\cdot(-\frac{3}{5})=-\frac{24}{25}[/tex][tex]\cos 2\theta=1-2(\frac{4}{5})^2=1-2\cdot\frac{16}{25}=1-\frac{32}{25}=-\frac{7}{25}[/tex]

The sum is:

[tex]\sin 2\theta+\cos 2\theta=-\frac{24}{25}-\frac{7}{25}=-\frac{31}{25}=-1.24[/tex]

Solve 1/3x + 4/9 = 7/9

Answers

We want to solve

[tex]\frac{1}{3}x+\frac{4}{9}=\frac{7}{9}[/tex]

We can subtract 4/9 on both sides.

[tex]\begin{gathered} \frac{1}{3}x+\frac{4}{9}-\frac{4}{9}=\frac{7}{9}-\frac{4}{9} \\ \\ \frac{1}{3}x=\frac{7}{9}-\frac{4}{9} \\ \\ \frac{1}{3}x=\frac{7-4}{9} \\ \\ \frac{1}{3}x=\frac{3}{9} \end{gathered}[/tex]

Now we have 3/9 on the right side, but we can simplify that fraction to 1/3

[tex]\begin{gathered} \frac{1}{3}x=\frac{3}{9} \\ \\ \frac{1}{3}x=\frac{1}{3} \end{gathered}[/tex]

And now we have the result!

[tex]x=1[/tex]

Therefore the final result is x = 1

Solve for x. Then find m (8x+4)°
(10x-6)°
Both lines are intersecting and the two equations are vertical pairs

Answers

For the vertical angles, the value of x is found as 5. The measure of the angle ∠QRT = 44° for the two intersecting lines.

What is referred as the vertical angles?When two lines intersect at a point, vertical angles are formed. They are always on equal footing. In other words, four angles are formed anytime two lines pass or intersect. We can see that two opposite angles are equal, and these are referred to as vertical angles.

For the given pair of angles in the question.

Two lines are intersecting to form two equations are vertical pairs.

∠QRT =  ∠VRS (vertical angles)

Put the values.

8x + 4 = 10x - 6

Simplifying.

2x = 10

x = 5

Put the values of 'x' in the angle.

∠QRT = 8x + 4

∠QRT = 8×5 + 4

∠QRT = 44

Thus, the measure of the angles ∠QRT is found as 44°.

To know more about the vertical angles, here

https://brainly.com/question/12162963

#SPJ10

Growing linearly, the balance owed on your credit card doubles from $600 to $1200 in 6 months. If the balance were growing according to the exponential function f(x)=600(1+0.1220)^x where x represents the number of months, what would the balance be after 6 months? Round your answer to the nearest cent.

Answers

Evaluate the given expression at x=6. This is, replace every x in the equation for 6 and solve:

[tex]\begin{gathered} f(6)=600(1+0.1220)^6 \\ f(6)=600(1.1220)^6 \\ f(6)=600\cdot(1.99506) \\ f(6)=1197.04 \end{gathered}[/tex]

The balance after 6 months is 1197.04.

In how many ways can 3person study groups beselected from a class of 25students?Note: nrn!r!(n-r)!nEnter

Answers

Answer:

2,300

Explanation:

This is given as:

[tex]\begin{gathered} 25\text{ combination 3 represented as:} \\ ^nC_r=\frac{n!}{r!(n-r)!} \\ n=25 \\ r=3 \\ ^{25}C_3=\frac{25!}{3!(25-3)!} \\ ^{25}C_3=\frac{25\times24\times23\times22!}{3!\times22!} \\ ^{25}C_3=\frac{25\times24\times23}{3\times2\times1} \\ ^{25}C_3=2300 \end{gathered}[/tex]

Therefore, there are 2,300 ways that 3 persons can be selected from 25 people

HELP ME!!!!!
One interior angle of a triangle is 95.5°, and the other two interior angles are congruent. What is the degree measure of one of the congruent angles?

42.25°
47.75°
84.5°
90°

Answers

The sum of all interior angles of a triangle is 180° thus the measure of one of the congruent angles is 42.25° so option (A) is correct.

What is a triangle?

A triangle is a 3-sided shape that is occasionally referred to as a triangle. There are 3 sides and three angles in every triangle, some of which may be the same.

Let's suppose the two congruent interior angles are x degrees.

It is known that the sum of all three angles inside a triangle will be 180°.

So,   m∠x+  m∠x +  95.5° = 180°

2m∠x = 180° - 95.5°

m∠x = 84.5/2 = 42.25°

Hence "The sum of all interior angles of a triangle is 180° thus the measure of one of the congruent angles is 42.25°".

For more about triangles,

https://brainly.com/question/2773823

 

#SPJ1  

A map has a scale of 1:2500. On the map a reservoir has an area of 2cm2.
what is the area of the reservoir? give your answer in m2

Answers

The actual area of the reservoir in m² as required in the task content is; 0.5m².

Conversions using scale factors.

It follows from the task content that the actual area of the reservoir is to be determined.

Hence, since it is given that the area of the reservoir on the map is 2cm² and the map has a scale of 1: 2500.

Hence, it follows from proportions that the area of the reservoir is; 2cm² × 2500 = 5000cm².

However, when expressed in m²; the actual area of the reservoir is; 5,000/10000 = 0.5m².

Ultimately, the area of the reservoir in m² is; 0.5m².

Read more on scale factor;

https://brainly.com/question/2826496

#SPJ1

Is my answer correct?

Answers

Answer:

Of course it is

Step-by-step explanation:

In the expression: 9 + 3y + 6 + y

Combining like terms, the expression becomes: 3y + y + 9 + 6

Final answer: 4y + 15 which is option B

So you're correct

Yes, you did it correctly


John and Ariana bought school supplies. John spent $10.30 on 5 notebooks and 7 pens. Ariana spent $7.20 on 4 notebooks and 4 pens. What is the cost of 1
notebook and what is the cost of 1 pen?

Answers

Answer:

$ 0.75 for one pen

$1.05 per notebook

Step-by-step explanation:

John and Ariana bought school supplies. John spent $10.30 on 5 notebooks and 7 pens. Ariana spent $7.20 on 4 notebooks and 4 pens. What is the cost of 1 notebook and what is the cost of 1 pen?

John: 5n + 7p = 10.5

let's solve for n:

5n + 7p = 10.5

subtract 7p from both sides:

5n + 7p -7p = 10.5 - 7p

5n = 10.5 - 7p

divide both sides by 5:

5n/5 = (10.5 - 7p)/5

n = 2.1 - 7/5 p

Ariana: 4n + 4p = 7.20

Let's substitute in: n = 2.1 - 7/5 p

4n + 4p = 7.20

4(2.1 - 7/5p) + 4p = 7.20

multiply left side:

8.4 - 5.6p + 4p = 7.2

subtract 8.4 from both sides:

8.4 - 5.6p + 4p - 8.4 = 7.2 - 8.4

- 5.6p + 4p = -1.2

combine p terms on left side:

-1.6p = -1.2

divide both sides by -1.6:

-1.6p/(-1.6) = -1.2/(-1.6)

p = 0.75 cents for one pen

Now solve to find the price of a notebook:

4n + 4p = 7.20 when p = 0.75

4n + 4(0.75) = 7.20

4n + 3 = 7.20

subtract 3 from both sides:

4n + 3 - 3 = 7.20 - 3

4n = 4.2

divide both sides by 4:

4n/4 = 4.2/4

n = 1.05 per notebook

CHECK: when n = 1,05 and p = 0.75

John: 5n + 7p = 10.5

5(1.05) + 7(0.75) = 10.5

10.5 = 10.5

Ariana: 4n + 4p = 7.20

4n + 4p = 7.20

4(1.05) + 4(0.75) = 7.20

7.2 = 7.2

Answer assumes no additional sales tax.

12 and 3/8 simplified

Answers

Answer:

(99/8)

Step-by-step explanation:

      3

12 -------

      8

8 × 12 = 96

96 + 3 = 99

 99

-------

  8

I hope this helps!

99/8

because 12x8=96 and 96+3=99 and it’s over 8

Solve the compound inequality -8≤x+4<5

Answers

Answer:

-12≤x<1

Step-by-step explanation:minus both side by 4

H-4/j=k for help
Please

Answers

Given:

The expression is 2p +5r = q.

The objective is to solve for p.

Explanation:

For the value of p, the expression can be solved as,

[tex]\begin{gathered} 2p+5r=q \\ 2p=q-5r \\ p=\frac{q-5r}{2} \end{gathered}[/tex]

Hence, the expression for p is, (q-5r)/2.

Part A answer and explanation pls​

Answers

Answer:

[tex]\frac{3}{13}[/tex]

Step-by-step explanation:

the fraction is formed by the number who prefer historical movies to the total preferring all movies.

total = 24 + 18 + 6 + 30 = 78

number preferring historical movies = 18

then fraction preferring historical movies is

[tex]\frac{18}{78}[/tex] ( divide numerator/ denominator by 6 )

= [tex]\frac{3}{13}[/tex] ← in simplest form

Answer:

From the Graph the total number of students who preferred historical movies =  18.

Similarly total number of  students = 24+18+6+30

                                                         = 78

Fraction of student who prefer Historical movies = 18/78

                                                                                = 3/13.

A number cube with faces labeled 1 to 6 is rolled once.
The number rolled will be recorded as the outcome.
Consider the following events.
Event A: The number rolled is less than 5.
Event B: The number rolled is even.
Give the outcomes for each of the following events.
If there is more than one element in the set, separate them with commas.
(a) Event "A or B": {0}
(b) Event "A and B": {0}
(c) The complement of the event A

Answers

According to the solving the probability are as follows:

a) Event "A or B": {1, 2, 3, 4, 6}

(b) Event "A and B": {2, 4}

(c) The complement of the event A: {5, 6}

Define the experiment's sample space and event:

The term "sample space" refers to a collection of probable results from a random experiment. The letter "S" indicates that this is the sample space. Events are a subset of what might happen in an experiment. Depending on the experiment, the outcomes in a sample area could change. In discrete or finite sample spaces, there are only a finite number of possible outcomes.

What is probability?

Simply put, probability measures how probable something is to occur. We can discuss the probabilities of various outcomes, or how likely they are, whenever we are unsure of how an event will turn out. Statistics is the study of events subject to probability.

According to the given data:

the sample space is, 1, 2, 3, 4, 5, 6 i.e., U = {1, 2, 3, 4, 5, 6}

Event A: The number rolled is less than 5

i.e., A = {1, 2, 3, 4}

Event B: The number rolled is even

i.e., B = {2, 4, 6}

Event A Or B A∪B = {1, 2, 3, 4} ∪ {2, 4, 6}

= {1, 2, 3, 4, 6}

Event "A and B"A∩B = {1, 2, 3, 4} ∩ {2, 4, 6}

= {2, 4}

Compliment of the event A

Comp(A) = U/ A = {1, 2, 3, 4, 5, 6} / {1, 2, 3, 4}

= {5, 6}

To learn more about events in probability visit:

https://brainly.com/question/11234923

#SPJ13

Triangle ABC is dilated by a scale factor of 4 to form triangle A’B’C

The coordinates of Vertex A’ are

The Coordinates of Vertex B’ are

The coordinates of Vertex C’ are

PLS HELP!!

Answers

The coordinates of vertex A is -2,1
The coordinates of vertex B is -3,3
The coordinates of vertex C is -1,3
my apologies if im incorrect but i think this the answer.

Find the x - and y-intercepts of the graph of the linear equation 3x - y = 2. The x-intercept is The y-interceptis

Answers

• x-intercept can be found by letting y equal to 0.

,

• y-intercept can be found by letting x equal to 0.

X intercept is the x-axis cutting point of the graph.

Y intercept is the y-axis cutting point of the graph.

The equation given is:

[tex]3x-y=2[/tex]

First, finding x intercept (let y = 0):

[tex]\begin{gathered} 3x-y=2 \\ 3x-0=2 \\ 3x=2 \\ x=\frac{2}{3} \end{gathered}[/tex]

Second, find y intercept (let x = 0):

[tex]\begin{gathered} 3x-y=2 \\ 3(0)-y=2 \\ 0-y=2 \\ y=-2 \end{gathered}[/tex]

Hence,

x-intercept: x = 2/3y-intercept: y = -2

P (0.3 < Z < 1.4) =0.30130.14030.91921.4014

Answers

Answer

P (0.3 < Z < 1.4) = 0.3013

Explanation

P (0.3 < Z < 1.4) = P (0 < Z < 1.4) - P (0 < Z < 0.3)

P (0.3 < Z < 1.4) = 0.4192 - 0.1179

P (0.3 < Z < 1.4) = 0.3013

i need tutor answer this two quedtion thabks you so much

Answers

5. Let the following inequality:

[tex]13-3m\text{ }<-2[/tex]

this is equivalent to:

[tex]-13+3m\text{ }>2[/tex]

this is equivalent to:

[tex]-13+13+3m\text{ }>2\text{ +13}[/tex]

this is equivalent to:

[tex]3m\text{ }>15[/tex]

solve for m:

[tex]m\text{ }>\frac{15}{3}=5[/tex]

that is :

[tex]m\text{ }>5[/tex]

6. Because of the graphic (real line), we can conclude that the correct interval would be:

[tex](-\infty,-3\rbrack\text{ = x}\leq-3[/tex]

Then we have to find the inequalities that have this solution interval.

a) Let the inequality

[tex]\frac{x}{3}+2\leq1[/tex]

this is equivalent to:

[tex]\frac{x}{3}+2-2\leq1-2[/tex]

this is equivalent to:

[tex]\frac{x}{3}\leq-1[/tex]

solve for x:

[tex]x\leq-3[/tex]

then. A) represent the graph.

b)Let the inequality

[tex]8-5x\ge23[/tex]

this is equivalent to:

[tex]-8+5x\leq-23[/tex]

this is equivalent to:

[tex]-8+8+5x\leq-23+8[/tex]

this is equivalent to:

[tex]5x\leq-15[/tex]

solve for x:

[tex]x\leq\frac{-15}{5}\text{ = -3}[/tex]

then the solution interval would be:

[tex]x\leq\text{-3}[/tex]

then. B) represent the graph.

c) Let the inequality:

[tex]-18\ge3+7x[/tex]

this is equivalent to:

[tex]-18+18\ge3+18+7x[/tex]

this is equivalent to:

[tex]0\ge21+7x[/tex]

this is equivalent to:

[tex]-21\ge7x[/tex]

solve for x:

[tex]x\text{ }\leq\frac{-21}{7}\text{ = -3}[/tex]

that is:

[tex]x\text{ }\leq\text{-3}[/tex]

then. C) represent the graph.

d) Let the inequality:

[tex]7x-3\leq18[/tex]

this is equivalent to:

[tex]7x-3+3\leq18+3[/tex]

this is equivalent to:

[tex]7x\leq21[/tex]

solve for x:

[tex]x\leq3[/tex]

We can conclude that this interval does not represent the graph because:

[tex]x\leq3\text{ }\ne\text{ }x\leq-3\text{ }[/tex]

Finally:

e) Let the inequality:

[tex]1-\frac{x}{2}\text{ }\leq2\text{ +}\frac{1}{2}[/tex]

this is equivalent to:

[tex]-1+\frac{x}{2}\text{ }\ge-2\text{ -}\frac{1}{2}[/tex]

this is equivalent to

[tex]-2+x\text{ }\ge-4\text{ -}1[/tex]

that is:

[tex]x\text{ }\ge-4\text{ -}1+2[/tex]

that is:

[tex]x\text{ }\ge-3[/tex]

THEN WE CAN CONCLUDE THAT THE CORRECT ANSWER ARE:

A), B), C) AND E)

Amy wants to save $500 to buy a TV. She saves $17 each week. The amount, A (in dollars), that she still needs after w weeks is given by the following function.
A (w) = 500 - 17w
(a). How much money does Amy still need after 7 weeks?
(b). If Amy still needs $211, how many weeks has she been saving?

Answers

After 7 weeks she would have the total of 119$

A. $381

B. 17 weeks

500-211=289/17=17

How do I simplify radicals in simplest radical form?

Answers

Expressing in simplest radical form just means simplifying a radical so that there are no more square roots, cube roots e.t.c

Examples of radicals are

[tex]\sqrt[]{4},\sqrt[2]{8},\sqrt[3]{16}[/tex]

For exmaple,

Given the radical

[tex]\sqrt[]{12}[/tex]

To simplify into the simplest radical,

Factorize the perfect square

[tex]\sqrt[]{12}=\sqrt[]{4\times3}[/tex]

Then we take out the pairs

[tex]\begin{gathered} \sqrt[]{12}=\sqrt[]{4}\times\sqrt[]{3} \\ \sqrt[]{12}=\sqrt[]{2\times2}\times\sqrt[]{3} \\ \sqrt[]{12}=\sqrt[]{2^2}\times\sqrt[]{3} \end{gathered}[/tex]

Simplify the result

[tex]\begin{gathered} \text{Where} \\ \sqrt[]{2^2}=2 \\ \sqrt[]{12}=2\times\sqrt[]{3} \\ \sqrt[]{12}=2\sqrt[]{3} \end{gathered}[/tex]

Hence, the simplified radical of the example used is

[tex]\sqrt[]{12}=2\sqrt[]{3}[/tex]

Algebra 14.11 Solve a system of equations using elimination: word problems NHRYou have prizes to reveal! GotWrite a system of equations to describe the situation below, solve using elimination, and fill inthe blanksThe administrative assistant at a software company often provides breakfast when there is amorning meeting. For last week's sales meeting, she purchased 6 dozen doughnuts and 2dozen croissants, spending a total of $42. In preparation for yesterday's safety meeting, shespent $40 on 2 dozen doughnuts and 5 dozen croissants. Assuming she purchased the itemsat the same bakery both times, how much does a dozen of each cost?A dozen doughnuts costs $and a dozen croissants costs $Submit

Answers

SOLUTION

Define a variable for the unkwons

[tex]\begin{gathered} \text{Let } \\ A\text{ dozen of doughnuts cost=\$x} \\ A\text{ dozen of Croissant cost=\$y} \end{gathered}[/tex]

Then

6 dozen of doughnuts and 2 dozen of croissant cost $42, is written as

[tex]6x+2y=42\ldots\text{equation 1}[/tex]

Similarly

$40 for 2 dozen of doughnuts and 5 dozen of croissant is witten as

[tex]2x+5y=40\ldots\text{equation 2}[/tex]

Applying Elimination to solve the two system of equation, we have

[tex]\begin{gathered} 6x+2y=42\ldots\text{equation 1} \\ 2x+5y=40\ldots\text{equation 2} \\ To\text{ eliminate x multiply equation 2 by 3 and equation 1 by 1} \\ 1\times(6x+2y=42)\rightarrow6x+2y=42 \\ 3\times(2x+5y=40)\rightarrow6x+15y=120 \end{gathered}[/tex]

Then, subtract the equation obtained above

[tex]\begin{gathered} 6x+2y=42 \\ 6x+15y=120 \\ -13y=-78 \\ \text{Divide both sides by -13} \\ -\frac{13y}{-13}=-\frac{78}{-13} \\ \\ y=6 \end{gathered}[/tex]

Hence Y=6

Then you Eliminate Y from eqaution 1 an d 2 by

Multiplying equation 1 by 5 and equation 2 by 2

[tex]\begin{gathered} 5\times(6x+2y=42)\rightarrow30x+10y=210 \\ 2\times(2x+5y=40)\rightarrow4x+10y=80 \end{gathered}[/tex]

The sunbtract the equation obtained

[tex]\begin{gathered} 30x+10y=210 \\ 4x+10y=80 \\ 26x=130 \\ \text{Divide both sides by 26} \\ \frac{26x}{26}=\frac{130}{26} \\ \\ x=5 \end{gathered}[/tex]

Hence X=5

Therefore

A Dozen of doughnuts cost $5

A Dozen of Croisant cost $6

(05.05 MC)A food truck did a daily survey of customers to find their food preferences. The data is partially entered in the frequency table. Complete thetable to analyze the data and answer the questions:Likes hamburgersboes not like hamburgers TotalLikes burritos41Does not like burritos54135Total110205Part A: What percentage of the survey respondents do not like both hamburgers and burritos? (2 points)Part & What is the marginal relative frequency of all customers that like hamburgers? (3 points)Part C Use the conditional relative frequencies to determine which data point has strongest association of its two factors. Use completesentences to explain your answer. (5 points)

Answers

EXPLANATION:

Given;

We are given a frequency table which displays a survey of numbers of customers that like hamburgers and burritos and those that do not like hamburgers and burritos.

Required;

We are required to analyze the table and use the values to answer the questions that follow.

Solution;

We shall begin by completing the table as follows;

Part A:

What percentage of the survey respondents do not like both burritos and hamburgers?

The percentage that do not like hamburgers is

[tex]Does\text{ }not\text{ }like\text{ }hamburgers=\frac{95}{205}[/tex]

The percentage that do not like burritos is

[tex]Does\text{ }not\text{ }like\text{ }burritos=\frac{135}{205}[/tex]

The percentage that does not like both burritos and hamburgers is;

[tex]\frac{54}{205}=0.263414634146[/tex]

Expressed as a percentage, this is

[tex]\begin{gathered} Percentage=0.263414634146\times100 \\ \\ Percentage=26.3414634146 \\ \\ Percentage=26.34\%\text{ }(rounded\text{ }to\text{ }the\text{ }nearest\text{ }hundredth) \end{gathered}[/tex]

The marginal relative frequency of all customers that like hamburgers is the total of all hamburger lovers divided by the total of all respondents.

[tex]\begin{gathered} Marginal\text{ }relative\text{ }frequency=\frac{hamburger\text{ }lovers}{total\text{ }respondents}=\frac{110}{205} \\ \\ Marginal\text{ }relative\text{ }frequency=0.536585365854 \\ \\ Marginal\text{ }relative\text{ }frequency=53.66\%\text{ }(rounded\text{ }to\text{ }2\text{ }decimal\text{ }places) \end{gathered}[/tex]

ANSWER:

Part (a) 26.34%

Part (b) 53.66%

Type the correct answer in each box.If matrix C represents (A − B) + A, the value of the entry represented by c41 is and the corresponding entry in (A + B) − A is .

Answers

Given the matrices A and B:

[tex]\begin{gathered} A=\begin{bmatrix}{-5} & {3} & {8} & {} \\ {3} & {6} & {-5} & {} \\ {5} & {-9} & {0} & {} \\ {7} & {3} & {4} & {}\end{bmatrix} \\ \\ B=\begin{bmatrix}{-7} & {-8} & {-5} & {} \\ {7} & {9} & {2} & {} \\ {2} & {5} & {-7} & {} \\ {2} & {8} & {-7} & {}\end{bmatrix} \end{gathered}[/tex]

We know that:

[tex]C=(A-B)+A=2A-B[/tex]

Then, using the matrices A and B:

[tex]\begin{gathered} C=2\cdot\begin{bmatrix}{-5} & {3} & {8} & {} \\ {3} & {6} & {-5} & {} \\ {5} & {-9} & {0} & {} \\ {7} & {3} & {4} & {}\end{bmatrix}-\begin{bmatrix}{-7} & {-8} & {-5} & {} \\ {7} & {9} & {2} & {} \\ {2} & {5} & {-7} & {} \\ {2} & {8} & {-7} & {}\end{bmatrix} \\ \\ C=\begin{bmatrix}{-10} & {6} & {16} & {} \\ 6 & {12} & {-10} & {} \\ 10 & {-18} & {0} & {} \\ 14 & 6 & 8 & {}\end{bmatrix}+\begin{bmatrix}{7} & {8} & {5} & {} \\ -{7} & -{9} & -{2} & {} \\ -{2} & {-5} & {7} & {} \\ {-2} & -{8} & {7} & {}\end{bmatrix} \\ \\ \therefore C=\begin{bmatrix}{-3} & 14 & 21 & {} \\ -1 & {3} & {-12} & {} \\ 8 & {-23} & 7 & {} \\ 12 & -2 & 15 & {}\end{bmatrix} \end{gathered}[/tex]

And the element C₄₁ (fourth row and first column) is:

[tex]C_{41}=12[/tex]

Now, for the matrix (A + B) - A = B:

[tex]B_{41}=2[/tex]

This graph shows the solutions to the inequalities y> 3x-2 and y<?x-10Does the system of inequalities have solutions? If so, which region containsthe solutions?108A84210-88441022 488BPopс-10A. There is a solution, and it is shown by region A.B. There is a solution, and it is shown by region C.C. There is no solution.D. There is a solution, and it is shown by region B.

Answers

Answer:

there is no solution (option C)

Explanation:

Given:

y > 3/2x - 2

y < 3/2x - 10

To find:

The solution to both graphs

To determine the solution of the graphs, we will consider their slopes

[tex]\begin{gathered} y\text{ > }\frac{3}{2}x\text{ - 2} \\ comparing\text{ with y = mx + b} \\ m\text{ = slope, b = y-intercept} \\ from\text{ the above, the slope = 3/2} \end{gathered}[/tex][tex]\begin{gathered} y\text{ < }\frac{3}{2}x\text{ - 10} \\ the\text{ slope = 3/2} \end{gathered}[/tex]

The slope of both inequalities is 3/2. If the slopes of two lines are the same, the lines are said to be parallel. This means both inequalities are parallel lines

For parallel lines, there is no solution because the lines do not intersect (meet).

Since both inequalities give parallel lines, there will be no solution (option C)

help me I'm practicing

Answers

we calculate the area of one trangle and multiply by 2

we apply formula of the triangle area

[tex]A=\frac{b\times h}{2}[/tex]

where b is the base and h the height of the triangle

then replacing

[tex]\begin{gathered} A=\frac{6\times5.2}{2} \\ \\ A=15.6 \end{gathered}[/tex]

area of one triangle is 15.6square centimeters

Area of both triangles

[tex]\begin{gathered} 15.6\times2 \\ =31.2 \end{gathered}[/tex]

Area of the two triangle bases is 31.2 square centimeters

a child is covering a square board with mosaic tiles the board is 40×40cn each tile is 2 cm ×2 cm how many mosaic tiles are needed for one complete now

Answers

Answer:

Step-by-step explanation:

The area of the floor is 4×3=12 sqare meters

=12×100×100= 120000 square centimetres.

Area of each tile is 20×20=400 square centimetres.

Therefore number of tiles required will be

120000÷400 =300.

300 tiles will be required to cover the floor.

Other Questions
please help me ty this needs a step-by-step explanation Assume a profitable economic well produces an average income of $23 million, while the average cost of a dry hole is $1.2 million. How much profit can be expected from a 100-well drilling program? assume that we have a function for an application of the form f(i, p), which gives the fraction of time that exactly i processors are usable given that a total of p processors is available. that means that nadia needs help running her bakery. nadia contacts her friend zoey, who has baking experience but lives in another state, and offers zoey a job. nadia orally promises zoey that she will employ zoey for at least two years. zoey quits her job in the other state, moves her family, and begins to work for nadia. after three months, nadia terminates zoeys employment. zoey sues nadia, claiming they had a contract for two years. if nadia defends the lawsuit by claiming the contract could not be enforced because it violates the statute of frauds, the court will likely hold in favor of: (note: we will learn about the statute of frauds in the next chapter.) what is -3.1 x (-2.9) Simplify the expression 9x^3/25y^2 and be sure to explain your steps! I need help with this Pleasee anyone explain other ways through which political parties compete for power during election A rectangle concrete patio is 20 FEET long, 12 FEET wide, & 4 INCHES thick. What is the volume in cubic feet, of the concrete? through: (-4,-2), slope 7/4 Pretend you are President of the United States. The Citizens and the economy of the USA need financial assistance to get back on their feet. List and describe three fiscal Policies you would initiate (describe each policy in 3 sentences): a 10-year semi-annual coupon bond with an $1000 par value pays an annual coupon rate of 6% and the market requires 8% apr. what is the appropriate coupon, time period, and discount rate respectively that needs to be used to correctly value this bond? group of answer choices $60, 10, Solve the rational equation for n The function is defined by h(x) = (4 + x)/(- 3 + 3x) Find h(4x) . After completing a repair on a vehicle what should the technician do next?Have the customer verify the repairPerform a quality inspectionReturn the vehicle to the lotMove to the next job In the 80s, 90s and early 2000s, the Chesapeake Bay was so polluted that fish and oyster populations were almost gone. In 2008, environmentalists were divided on how to fix the bay. Some demanded widespread seeding of sterile imported oysters to deal with worsening water quality that was wiping out aquatic life in the bay. Adding the imported oysters could slowly filter and clean the bay water. Others warned that this could be a disaster because a few of the imported oysters would be fertile and might proliferate, forcing out the last of the Eastern oysters or interbreeding with the native species. (Source: 1999-2020 National Center for Case Study Teaching in Science, University at Buffalo.)If local officials decide to place imported oysters in bay, how could this change the capacity of the bay to support human activity? ASAP!! 4(x + 4) + 2x = 52 Solve for X and include step by step explanation for branliest. 80/15 y 6 son equivalentes?6 y 5 7/8 son equivalentes? A hollow, spherical shell with mass 2.00 kg rolls without slipping down a 33.0 slope.Find the acceleration.Find the friction force. In early 2016, the United Nations released a report on the biodiversity of pollinators around the world. According to the report, two out of five species of invertebrate pollinators (primarilybees, but also butterflies and other insects) now face extinction, and one in six of the vertebrate pollinators (birds) do as well. Four causes have been identified for the loss of biodiversity:loss of basic habitat, simplification of the landscape to promote single crops, indiscriminate use of pesticides, and the urbanization of landscapes. Which of the following activities would helprestore pollinator populations in the United States?A.Requiring farmers to produce wheat instead of corn or potatoesB.Encouraging farmers to enhance their farms by planting flowering hedgerows and natural nesting sites for pollinatorsC.Making it illegal to produce or sell honey products within the United StatesD.Requiring farmers to use pesticides all year long, not just when flowers are blooming